2023 AMC 12A Problems/Problem 20

Revision as of 22:42, 9 November 2023 by Lptoggled (talk | contribs)

Problem

Rows 1, 2, 3, 4, and 5 of a triangular array of integers are shown below.

Solution 1

First, let $R(n) be the sum of the$n$th row. Now, with some observations and math instinct, we can guess that$R(n) = 2^n - n$now we try to prove it by induction,$R(1) = 2^n - n = 2^1 - 1 = 1$(works for base case!)$R(k) = 2^k - k$$ (Error compiling LaTeX. Unknown error_msg)R(k+1) = 2^{k+1} - (k + 1) = 2(2^k) - k - 1$Now by definition from the question, the next row is always: double the sum of last row (Each number from last row branches off toward left and right to the next row), plus # of new row, minus 2 (leftmost and rightmost are just 1)$ (Error compiling LaTeX. Unknown error_msg)2(2^k - k) + (k + 1) - 2 = 2(2^k) - k - 1$Hence proven.

Simply substitute$ (Error compiling LaTeX. Unknown error_msg)n = 2023$, we get$R(2023) = 2^2023 - 2023$

Last digit of 2^2023 is 8, 8-3 = \boxed{\textbf{(C)} 5}

~lptoggled

See also

2023 AMC 12A (ProblemsAnswer KeyResources)
Preceded by
Problem 19
Followed by
Problem 21
1 2 3 4 5 6 7 8 9 10 11 12 13 14 15 16 17 18 19 20 21 22 23 24 25
All AMC 12 Problems and Solutions

The problems on this page are copyrighted by the Mathematical Association of America's American Mathematics Competitions. AMC logo.png